Question

Suppose a company has the following total cost function: TC = 20 + 3x + 54x (3) Where TC represents total cost (in $) and X i

0 0
Add a comment Improve this question Transcribed image text
Know the answer?
Add Answer to:
Suppose a company has the following total cost function: TC = 20 + 3x + 54x...
Your Answer:

Post as a guest

Your Name:

What's your source?

Earn Coins

Coins can be redeemed for fabulous gifts.

Not the answer you're looking for? Ask your own homework help question. Our experts will answer your question WITHIN MINUTES for Free.
Similar Homework Help Questions
  • Assume that a competitive firm has the total cost function: TC=1q3−40q2+740q+1500 Suppose the price of the...

    Assume that a competitive firm has the total cost function: TC=1q3−40q2+740q+1500 Suppose the price of the firm's output (sold in integer units) is $750 per unit. Using calculus and formulas (but no tables and restricting your use of spreadsheets to implementing the quadratic formula) to find a solution, what is the total profit at the optimal integer output level? Please specify your answer as an integer. Hint 1: The first derivative of the total cost function, which is cumulative, is...

  • 2. Consider the following cost minimization problem. A firm minimizes total cost given by, TC =...

    2. Consider the following cost minimization problem. A firm minimizes total cost given by, TC = wL+rK subject to an output constraint as given by the production function, y=f(K,L)=8K05 +420S, where TC refers to total cost, L is labor input, K is capital input, r is the price of capital, w is the wage rate, and y is output. a. Derive the factor demand functions and the optimal cost function. The first order conditions and all the steps involved in...

  • Q: Suppose a firm's total cost function is TC = 16 + 5Q + 4Q2 ....

    Q: Suppose a firm's total cost function is TC = 16 + 5Q + 4Q2 . What is the output level that minimizes average total cost?

  • Suppose that each firm in a competitive industry has the following costs: Total Cost: TC =...

    Suppose that each firm in a competitive industry has the following costs:Total Cost: TC=50+1/2 q2Marginal Cost: MC=qwhere q is an individual firm's quantity produced.The market demand curve for this product is:Demand QD=160-4 Pwhere P is the price and Q is the total quantity of the good.Each firm's fixed cost is $_______ What is each firm's variable cost?1/2 q50+1/2 q1/2 q^{2}qWhich of the following represents the equation for each firm's average total cost?50/q+1/2 q50+1/2 q50/q1/2 qComplete the following table by computing the...

  • suppose a firm's total cost of production (TC) is tc=2Q^2 mc=4Q Suppose a firm's total cost...

    suppose a firm's total cost of production (TC) is tc=2Q^2 mc=4Q Suppose a firm's total cost of production (TC) is TC = 20 MC = 40 What do the firm's average total cost curve, average variable cost curve, and marginal cost curve look like? Draw the following curves from 0 to 5 units of output. 1.) Using the line drawing tool.graph the firm's average total cost curve and label it ATC 2.) Using the line drawing tool, graph the firm's...

  • Suppose that a firm has a short run, total cost function given by: TC= 1089 +10q...

    Suppose that a firm has a short run, total cost function given by: TC= 1089 +10q +9q2. 1. Determine the profit-maximizing quantity of production when price is $244. _____________________________________ q= 13 2. Calculate the price at which this firm breaks even (i.e. profit = $0). _____________________________________ $208 3. Calculate the price at which this firm shuts down in the short run. _____________________________________ $10 The answers are given but can you show how to get them step by step.

  • Suppose that a price searcher had a total cost function given by: TC= 20 + 2q...

    Suppose that a price searcher had a total cost function given by: TC= 20 + 2q +0.25Q?The demand for the price searcher's product is given by: Qp= 100 -5P Calculate the profit-maximizing quantity the monopolist will produce. Answer: Check

  • Suppose a firm has the following total cost function: TC = 300 + 40 Q –...

    Suppose a firm has the following total cost function: TC = 300 + 40 Q – 8Q2 + (2/3) Q3 (a) Write an equation for (i) average fixed cost; and (ii) average variable cost (b) What will be the value of average (total) cost when Q = 60? (c) What will be the marginal cost, when Q = 20? (d) For this firm what will be the value of average variable cost at its minimum

  • Suppose that the long-run total cost function for the typical mushroom producer is given by: TC=wq2...

    Suppose that the long-run total cost function for the typical mushroom producer is given by: TC=wq2 -10q+100 Where q represents the output of the typical firm and w represents the hourly wage rate mushroom pickers. The market for mushrooms is perfectly competitive. Suppose also that the demand for mushrooms is given by Q= -1000p+35,000 Where Q is the total quantity demand and P is the market price of mushrooms. 1.   If the wage rate for mushroom pickers is $4.00, what...

  • Suppose the firm's total cost and marginal cost functions are given by TC = 54 +...

    Suppose the firm's total cost and marginal cost functions are given by TC = 54 + Q + 2Q3 and MC = 1 + 4Q2, respectively. What is the output level that minimizes average total cost? 2 3 6 8

ADVERTISEMENT
Free Homework Help App
Download From Google Play
Scan Your Homework
to Get Instant Free Answers
Need Online Homework Help?
Ask a Question
Get Answers For Free
Most questions answered within 3 hours.
ADVERTISEMENT
ADVERTISEMENT
ADVERTISEMENT